What is the Algebraic Method for Finding Delta?

  • Thread starter Thread starter Saladsamurai
  • Start date Start date
  • Tags Tags
    Delta
Click For Summary
SUMMARY

The Algebraic Method for Finding Delta involves determining the delta value for the function f(x) = x² at the limit L = 4, centered around x₀ = -2 with an epsilon (e) of 0.5. The critical inequality derived is -0.5 < x² - 4 < 0.5, leading to the bounds 3.5 < x² < 4.5. The correct interpretation requires careful consideration of the square roots, yielding the condition |x - x₀| < √3.5 - (-2) ≈ 3.87. The confusion arises from the need to clarify whether to use positive or negative square roots in the context of the given x₀.

PREREQUISITES
  • Understanding of limits in calculus
  • Familiarity with the epsilon-delta definition of limits
  • Knowledge of square root properties
  • Ability to manipulate inequalities
NEXT STEPS
  • Study the epsilon-delta definition of limits in detail
  • Learn how to apply the Algebraic Method for different functions
  • Practice solving limit problems involving square roots
  • Explore common pitfalls in limit calculations and how to avoid them
USEFUL FOR

Students of calculus, mathematics educators, and anyone seeking to deepen their understanding of limit concepts and the Algebraic Method for Finding Delta.

Saladsamurai
Messages
3,009
Reaction score
7
Given f(x) = x2, L = 4, xo = -2, e = 0.5 find delta.

-0.5 < x2 - 4 < 0.5

3.5 < x2 < 4.5

(3.5)1/2 - (-2) < x - (-2) < (4.5)1/2 - (-2)

=>|x - xo| < (3.5)1/2 - (-2) ~ 3.87

My text says the answer is 0.12 ?

I was convinced that I have been doing these right. Am I?
 
Physics news on Phys.org
Saladsamurai said:
Given f(x) = x2, L = 4, xo = -2, e = 0.5 find delta.

-0.5 < x2 - 4 < 0.5

3.5 < x2 < 4.5

you have a skipped an important step here, when you take the square root, do you take the positive or negative squareroot... thoughts?

ie. assuming
a2 < x2 < b2
you need to be careful about whether you treat a & b as positive or negative numbers


Saladsamurai said:
(3.5)1/2 - (-2) < x - (-2) < (4.5)1/2 - (-2)

=>|x - xo| < (3.5)1/2 - (-2) ~ 3.87

My text says the answer is 0.12 ?

I was convinced that I have been doing these right. Am I?

As a test try using you delta & see what e it gives you?

As an aside I would tray & start a little more explicit:
so there exists d>0 such that for all |x-x0|< d, |f(x)-f(x0)|<e

Now
|f(x)-f(x0)|= |x^2-x0^2|= |(x+x0)(x-x0)| < e

Any ideas how to re-write this in terms of delta?
 
Last edited:
You did a mistake.

It should be:

√3.5 < |x| < √4.5
 
lanedance said:
As an aside I would tray & start a little more explicit:
so there exists d>0 such that for all |x-x0|< d, |f(x)-f(x0)|<e

Now
|f(x)-f(x0)|= |x^2-x0^2|= |(x+x0)(x-x0)| < e

Okay then! :smile:

I am not really sure what you are doing here? Could you please explain?

Thanks :)
 
Starting with your inequality 3.5 < x2 < 4.5, it must be either that
1) \sqrt{3.5} < x < \sqrt{4.5}, OR
2) -\sqrt{4.5} < x < -\sqrt{3.5}

Since x0 in your problem is -2, which of the inequalities above is the one that should be used for this problem? For your limit, x should be reasonably close to x0.
 
Question: A clock's minute hand has length 4 and its hour hand has length 3. What is the distance between the tips at the moment when it is increasing most rapidly?(Putnam Exam Question) Answer: Making assumption that both the hands moves at constant angular velocities, the answer is ## \sqrt{7} .## But don't you think this assumption is somewhat doubtful and wrong?

Similar threads

  • · Replies 9 ·
Replies
9
Views
4K
  • · Replies 31 ·
2
Replies
31
Views
2K
  • · Replies 5 ·
Replies
5
Views
2K
  • · Replies 31 ·
2
Replies
31
Views
4K
Replies
3
Views
2K
  • · Replies 3 ·
Replies
3
Views
1K
  • · Replies 3 ·
Replies
3
Views
3K
Replies
8
Views
1K
Replies
2
Views
1K
Replies
1
Views
2K